Foro de preguntas y respuestas de Matemáticas

logo beUnicoos
Los foros de unicoos se han unificado en nuestra nueva plataforma beUnicoos. Para dejar nuevas preguntas deberás hacerlo allí, donde además podrás encontrar nuevas asignaturas y herramientas para ayudarte más con tus estudios.

  • icon

    Laura
    el 13/8/19

    Buenod días, me pide que encuentre el valor de x pero de me complicó mucho, me ayudarían? Gracias

    replythumb_up0 voto/sflag
    icon

    Antonio Silvio Palmitano
    el 13/8/19

    Vamos con una orientación.

    Distribuyes los denominadores en todos los términos fraccionarios, y queda:

    d/(b+c) - ax/(b+c) + d/(a+c) - bx/(a+c) + d/(a+b) - cx/(a+b) = -d/(a+b+c) + 4x;

    luego, restas d/(b+c), restas d/(a+c), restas d/(a+b) y restas 4x en ambos miembros, y queda:

    -ax/(b+c) - bx/(a+c) - cx/(a+b) - 4x = -d/(a+b+c) - d/(b+c) - d/(a+c) - d/(a+b), multiplicas por -1 en todos los términos, y queda:

    ax/(b+c) + bx/(a+c) + cx/(a+b) + 4x = d/(a+b+c) + d/(b+c) + d/(a+c) + d/(a+b), extraes factor común en ambos miembros, y queda:

    x*( a/(b+c) + b/(a+c) + c/(a+b) + 4 ) = d*( 1/(a+b+c) + 1/(b+c) + 1/(a+c) + 1/(a+b) ) (1).

    Luego, tienes la expresión del agrupamiento del primer miembro:

    A = a/(b+c) + b/(a+c) + c/(a+b) + 4, extraes denominador común, y queda:

    A = ( a(a+c)(a+b) + b(b+c)(a+b) + c(b+c)(a+c) + 4(b+c)(a+c)(a+b) ) / (b+c)(a+c)(a+b) (2).

    Luego, tienes la expresión del agrupamiento del segundo miembro:

    B = 1/(a+b+c) + 1/(b+c) + 1/(a+c) + 1/(a+b), extraes denominador común, y queda:

    B = ( (b+c)(a+c)(a+b) + (a+b+c)(a+c)(a+b) + (a+b+c)(b+c)(a+b) + (a+b+c)(b+c)(a+c) ) / (a+b+c)(b+c)(a+c)(a+b) (3).

    Luego, sustituyes la expresión señalada (2) en el primer miembro, sustituyes la expresión señalada (3) en el segundo miembro, todo en la ecuación señalada (1), y queda:

    x*( ( a(a+c)(a+b) + b(b+c)(a+b) + c(b+c)(a+c) + 4(b+c)(a+c)(a+b) ) / (b+c)(a+c)(a+b) ) = d*( ( (b+c)(a+c)(a+b) + (a+b+c)(a+c)(a+b) + (a+b+c)(b+c)(a+b) + (a+b+c)(b+c)(a+c) ) / (a+b+c)(b+c)(a+c)(a+b);

    luego, multiplicas por (b+c)(a+c)(a+b) en ambos miembros, simplificas, y queda:

    x*( a(a+c)(a+b) + b(b+c)(a+b) + c(b+c)(a+c) + 4(b+c)(a+c)(a+b) ) = d*( ( (b+c)(a+c)(a+b) + (a+b+c)(a+c)(a+b) + (a+b+c)(b+c)(a+b) + (a+b+c)(b+c)(a+c) ) / (a+b+c);

    luego, divides en ambos miembros por ( a(a+c)(a+b) + b(b+c)(a+b) + c(b+c)(a+c) + 4(b+c)(a+c)(a+b) ), y finalmente queda:

    x = d*( ( (b+c)(a+c)(a+b) + (a+b+c)(a+c)(a+b) + (a+b+c)(b+c)(a+b) + (a+b+c)(b+c)(a+c) ) / (a+b+c)( a(a+c)(a+b) + b(b+c)(a+b) + c(b+c)(a+c) + 4(b+c)(a+c)(a+b) ),

    y observa que no distribuimos términos en el numerador y el denominador de la expresión remarcada, a fin de no dilatar más esta tarea ardua.

    Espero haberte ayudado.

    thumb_up0 voto/sflag
  • icon

    Jose
    el 13/8/19

    Como se puede realizar ese ejercicio?,Muchas gracias

    replythumb_up0 voto/sflag
    icon

    César
    el 13/8/19


    thumb_up1 voto/sflag
  • icon

    Jose
    el 13/8/19

     Hola queria saber si este ejercicio se puede realizar sin calculadora, y si es que se puede ,como se haria?,Muchas gracias

    replythumb_up0 voto/sflag
    icon

    César
    el 13/8/19


    thumb_up1 voto/sflag
  • icon

    Jose Ramos
    el 13/8/19

    Hola amigos.

    Alguien sabe como resolver esta ecuación sin hacerlo "a ojo" (las soluciones son fáciles de obtener a simple vista: 0,π, -π). El problema es si conocéis algún procedimiento  para resolverlo sin utilizar los métodos de resolución de ecuaciones por aproximación:

    1 + cos x = (-2x22 )+2

    replythumb_up0 voto/sflag
    icon

    Antonio Silvio Palmitano
    el 14/8/19

    Observa que el segundo término del primer miembro es trigonométrico, y que el segundo miembro es polinómico,

    por lo que estás tratando con una ecuación que requiere métodos avanzados para ser resuelta, como puede ser por aproximaciones, desarrollos de Tayloru otros métodos numéricos, de ahí que la resuelven por medio del método de inspección, o sea, a simple vista.

    Espero haberte ayudado.

    thumb_up1 voto/sflag
  • icon

    Leilyta Banegas
    el 13/8/19

    Hola! Alguien me puede ayudar con esos ejercicios que están marcados... Hay que demostrarlos:

    Desde ya muchas gracias!

    Saludos Leily

    replythumb_up0 voto/sflag
    icon

    Jose Ramos
    el 13/8/19

    Son ilegibles. Haz un enfoque más directo al ejercicio en cuestión

    thumb_up0 voto/sflag
    icon

    César
    el 13/8/19


    thumb_up0 voto/sflag
  • icon

    Diego Tapia
    el 13/8/19

    Hola me podrían ayudar con este ejercicio? (El 23)

    replythumb_up0 voto/sflag
    icon

    Jose Ramos
    el 13/8/19


    thumb_up0 voto/sflag
    icon

    Jose Ramos
    el 13/8/19


    thumb_up0 voto/sflag
    icon

    César
    el 13/8/19


    thumb_up0 voto/sflag
  • icon

    Laura
    el 13/8/19

    Hola, se me dificultó resolver este ejercicio, me ayudarían? Gracias

    replythumb_up0 voto/sflag
    icon

    César
    el 13/8/19

    Que te piden en el ejercicio?


    thumb_up0 voto/sflag
    icon

    Laura
    el 13/8/19

    Encontrar el valor de x.

    thumb_up0 voto/sflag
  • icon

    mefesto pat
    el 13/8/19

    Tiene sentido?

    replythumb_up0 voto/sflag
    icon

    Jose Ramos
    el 13/8/19

    Tiene sentido y la igualdad es cierta. Lo he demostrado partiendo de que w puede tomar dos valores (complejos por supuesto) que no son 1:  (1,120º)  y (1,240º). En ambos casos, y sustituyendo por w esos dos valores, el determinante es 3.

    thumb_up0 voto/sflag
    icon

    Antonio Silvio Palmitano
    el 14/8/19

    Desarrollas el determinante (nosotros lo hacemos según su primera fila, pero puedes emplear la Regla de Sarrus, la Regla de la Estrella, u otro método válido, si prefieres), y queda:

    D = 1*(1 - w2) - 1*(1 - w3) + w2*(w - w2),

    resuelves coeficientes en los dos primeros términos, extraes factor común (w) en el agrupamiento del tercer término, y queda:

    D = (1 - w2) - (1 - w3) + w3*(1 - w),

    extraes factor común (-1) en el segundo término (observa que cambia su signo), y queda:

    D = (1 - w2) + (-1 + w3) + w3*(1 - w),

    conmutas términos en el agrupamiento que tienes en el segundo término, y queda:

    D = (1 - w2) + (w3 - 1) + w3*(1 - w),

    reemplazas el valor correspondiente a la expresión del segundo término (observa que es igual a cero tal como sugieren en tu enunciado), y queda:

    D = (1 - w2) + 0 + w3*(1 - w),

    cancelas el término nulo, y queda:

    D = (1 - w2) + w3*(1 - w),

    factorizas el primer término (observa que es una resta de cuadrados perfectos), y queda:

    D = (1 - w)*(1 + w) + w3*(1 - w),

    reemplazas el valor del primer factor del último término (observa que tienes en tu enunciado: w3 = 1), y queda:

    D = (1 - w)*(1 + w) + 1*(1 - w),

    extraes factor común ( (1 - w) ), y queda:

    D = (1 - w)*(1 + w + 1),

    reduces términos semejantes en el segundo factor, y queda:

    D = (1 - w)*(2 + w) (*).

    Luego, considera las expresiones polares de las dos soluciones de la ecuación: w3 = 1 que son distintas de 1 que indica el colega José, y tienes dos opciones:

    1°)

    w = 1120°, que en forma trigonométrica queda expresada:

    w = 1*( cos(120°) + i*sen(120°) ), distribuyes, reemplazas los valores exactos en cada término, y queda:

    w = -1/2 + √(3)*i/2;

    luego, planteas las expresiones de los factores de la ecuación señalada (1), y queda:

    1 - w = 1 - (-1/2 + √(3)*i/2) = 1 + 1/2 - √(3)*i/2 = 3/2 - √(3)*i/2 (1a),

    2 + w = 2 + (-1/2 + √(3)*i/2) = 2 - 1/2 + √(3)*i/2) = 3/2 + √(3)*i/2 (1b);

    luego, reemplazas las expresiones numéricas señaladas (1a) (1b) en la ecuación señalada (*), y queda:

    D = (3/2 - √(3)*i/2)*(3/2 + √(3)*i/2), desarrollas (observa que tienes una resta de dos términos multiplicada por su suma), y queda:

    D = (3/2)2 - (√(3)*i/2)2, resuelves ambos términos (presta atención al segundo término), y queda:

    D = 3/2 - (-3/2), resuelves, y queda:

    D = 3.

    2°)

    w = 1240°, que en forma trigonométrica queda expresada:

    w = 1*( cos(240°) + i*sen(240°) ), distribuyes, reemplazas los valores exactos en cada término, y queda:

    w = -1/2 - √(3)*i/2;

    luego, planteas las expresiones de los factores de la ecuación señalada (1), y queda:

    1 - w = 1 - (-1/2 - √(3)*i/2) = 1 + 1/2 + √(3)*i/2 = 3/2 + √(3)*i/2 (2a),

    2 + w = 2 + (-1/2 - √(3)*i/2) = 2 - 1/2 - √(3)*i/2) = 3/2 - √(3)*i/2 (2b);

    luego, reemplazas las expresiones numéricas señaladas (2a) (2b) en la ecuación señalada (*), y queda:

    D = (3/2 + √(3)*i/2)*(3/2 - √(3)*i/2), desarrollas (observa que tienes una suma de dos términos multiplicada por su resta), y queda:

    D = (3/2)2 - (√(3)*i/2)2, resuelves ambos términos (presta atención al segundo término), y queda:

    D = 3/2 - (-3/2), resuelves, y queda:

    D = 3.

    Espero haberte ayudado.


    thumb_up0 voto/sflag
  • icon

    Elizabeth Jovi
    el 13/8/19

    Please  .si pudieran ayudarme. 🙏Se los agradecería. 

    replythumb_up0 voto/sflag
    icon

    Martin
    el 13/8/19

    4) A

    5) B 

    6) C (En matematicas, un numero primo es un numero natural mayor que 1, que tiene unicamente dos divisores, por uno y por si mismo)

    thumb_up1 voto/sflag
  • icon

    Elizabeth Jovi
    el 13/8/19

    Soy nula en matemáticas,  nunca me gustó,  y me cuesta entenderla , si pudieran ayudarme. Please🙏

    replythumb_up0 voto/sflag
    icon

    Martin
    el 13/8/19

    9) B (simplemente hay que dividir por 2 tanto arriba como abajo en la fraccion) = 3/4

    10) A (ya que hay una mayor cantidad es mas probable que saque del sabor frambuesa)

    thumb_up1 voto/sflag